Identify the initial condition in the table.

Identify The Initial Condition In The Table.

Answers

Answer 1

Answer:

i had the answer but lost it ill look into more for u

Step-by-step explanation:

Answer 2

The given table can be represented as :

y = -x

And here, initial condition is 0


Related Questions

A box of chocolates contains bars of milk chocolate and dark chocolate. The weight of each bar of milk chocolate and dark chocolate is 150 ounces and 100 ounces respectively. If the number of bars of dark chocolate is 12 more than the number of bars of milk chocolate, and the total weight of the chocolates in the box is 2950 ounces, find the number of bars of milk and dark chocolate.

Answers

Answer:

7 milk chocolate

19 dark chocolate

Step-by-step explanation:

It is 2950 ounces in total, and dark chocolate is 100 ounces per bar and milk chocolate is 150 ounces per bar. There are 12 more dark chocolate bars so I multiplied 12 by 100 and that was 1,200. Then I took 2,950-1200=1,750. From there I subtracted 150 and 100 till I got to 0.

Number of bars of milk chocolate: 17

Number of bars of dark chocolate: 29

Let's use algebraic equations to solve the problem. Let's denote the number of bars of milk chocolate as "x" and the number of bars of dark chocolate as "y."

1. Weight equation: The total weight of the chocolates in the box is 2950 ounces.

We can express this as an equation:

150x + 100y = 2950  

2. Quantity equation: The number of bars of dark chocolate is 12 more than the number of bars of milk chocolate.

We can express this relationship as:

y = x + 12

Now, we have a system of two equations with two variables:

Equation 1 : 150x + 100y = 2950

Equation 2 : y = x + 12

To find the values of x and y, we can use substitution or elimination. Let's use substitution in this case.

From Equation 2, we can substitute the value of "y" in terms of "x" into Equation 1:

150x + 100(x + 12) = 2950

Now, we can solve for "x":

250x + 1200 = 2950

250x = 1750

x = 7

Now that we have the value of "x," we can find the value of "y" using Equation 2:

y = x + 12

y = 7 + 12

y = 19

So, the number of bars of milk chocolate (x) is 7, and the number of bars of dark chocolate (y) is 19.

To know more about Algebraic Equations here

https://brainly.com/question/29964992

#SPJ2

Enter numbers in the boxes to write an equation that is equivalent to 3/4b−2/3b=−9
Please help I don't know what to do!!!!!

Answers

Answer:

3/4b−2/3b=−9 = -108. so, i just used equivalent fractions of both of those numbers :D

Step-by-step explanation:

6/8b - 4/6b = -9

anthony runs a bicycle courier company that charges $3 for delivery plus $0.50 per mile. Which equation matches this relationship

Answers

Answer:

c = .5m + 3

Step-by-step explanation:

This graph shows a reciprocal parent function.
Which statement best describes the function
A. The function is always increasing
B. The function is increasing when x<0
C. The function is never increasing
D. The function is increasing when x>0

Answers

Answer:

C. The function is never increasing

Step-by-step explanation:

According to the graph, the line goes down in both sections as the value of x increases. It means the function is always decreasing.

Correct answer choice is:

C. The function is never increasing

The best describes the function is,

''The function is never increasing.''

What is mean by Function?

A relation between a set of inputs having one output each is called a function. and an expression, rule, or law that defines a relationship between one variable (the independent variable) and another variable (the dependent variable).

Given that;

This graph shows a reciprocal parent function.

Since, We have;

According to the graph, the line goes down in both sections as the value of x increases.

It means the function is always decreasing.

Hence, Correct answer choice is:

C. The function is never increasing.

Learn more about the function visit:

https://brainly.com/question/11624077

#SPJ7

PLEASE SOMEONE ANSWERRR I have school in 20 minutes please help D:

Answers

ok i gotchu

1.) 4 1/4

2.)6 2/3

3.)11

4.)3 11/15

5.)1 1/8

6.)1 17/18

7.)4

8.)4 1/12

9.)12 5/6

A farmer is building a fence to enclose a rectangular area against an existing wall, shown in the figure below.

Three of the sides will require fencing and the fourth wall already exists.

If the farmer has 152 feet of fencing, what are the dimensions of the region with the largest area?

Answers

Answer:

where is the figure?? thanks

Line l is the perpendicular bisector of segments ps. Unfortunately, s got lost


Mark point s

Answers

The perpendicular bisector of a line divides the line into two equal segments and is perpendicular to the line

The perpendicular distance of point P from the line l is equal to the distance of the lost point s from the line l and points P and s are on opposite sides of the line l

The point s can be marked by constructing the perpendicular to line l passing through P

The steps to construct a perpendicular line from a point to a line and mark point s are given as follows;

Place the compass at the point P, and draw an arc intersecting line l at two points, A and BPlace the compass at point A and draw an arc with a radius, r, larger than half the length of segment AB above and below line lPlace the compass point at point B, and with the same radius r draw arcs intersecting the the arcs drawn from point A above and below the lineMark the point of intersection of the two arcs above and below the line l as the points D and EJoin points D and E with the a line to form the line DE drawn with a straightedge and extend DE line such that it passes through point PMark the point of intersection of the line DE and line l as the point F on the line lPlace the compass at the point F, and with a radius FP, draw an arc to intersect the line DE on the other side of the line lMark the point of intersection of the line DE by the arc of radius FP as the point s

The constructed point s above is the required point and points P and s are located on opposite sides of the line l

Learn more here:

https://brainly.com/question/14261048

Helppppp plzzzzzzzzzzz

Answers

So how you can solve this is by using some cool tricks.

Look at the end number of computers, which is 40 right?
So with the total number of 40, you will simply subtract 10 and 5, to get the missing number.
Which is 40-10-5= 25

Then for printers you would do, 8-2= 6
Then by filling in the first blank you can move into the second blank by adding up all the three numbers you now have,
8+2+6= 16

Add −1.5+1.75

Plot the first addend and the sum on the number.

Answers

Answer:

0.5+0.75=0.80

now

1+1=2

2+0.80=2.89

Answer:

.25

Step-by-step explanation:

its very easy btw you shouldn't have needed to post it just use a calculator.

Write the domain and range of the function represented below. (2,9), (3, 4), (-1,2), (6, 1)

Domain:
Range:​

Answers

Answer:

Domain:  { 2 ,  3 ,  − 1 ,  6 }

Range: { 9 ,  4 ,  2 ,  1 }

Step-by-step explanation:

   

Please help!
Solve for x
3(x - 4) - 2x = 6

Answers

Answer:

Step-by-step explanation:

first open the bracket

3x-12-2x=6

collect like terms

3x-2x-12=6

x-12=6

x=12+6

x=18

This is your answer

A sandwich store charges a delivery fee to bring lunch to an office building. One office pays 33$ for 4 turkey sandwiches. Another office pays 61$ for 8 turkey sandwiches. How much does each turkey sandwich cost?

Answers

Answer:

8.25 for the 33$ and aporximately 7.63 for the 61$

Step-by-step explanation:

calculators exist

4. Identify the graph of the solution set of 9 > 3 + 2x.

Answers

Answer:

A

Step-by-step explanation:

9 > 3 + 2x

6 > 2x

3 > x

or

x < 3

and that means 3 is NOT included (otherwise there must be a smaller or equal sign). hence A is the right answer.

The graph of the solution set [tex]9 > 3 + 2x[/tex]  is represented by the inequality relation x < 3.

The correct answer is Option A.

Given data:

The inequality is represented as 9 > 3 + 2x

The solution set of the inequality 9 > 3 + 2x can be found by isolating x on one side of the inequality:

Subtract 3 from both sides: 9 - 3 > 2x

Simplify: 6 > 2x

Divide both sides by 2: 3 > x

So, the solution set is x < 3.

Graphically, this represents all the values of x that are to the left of (less than) 3 on the number line. It forms an open interval to the left of 3. The graph consists of all real numbers less than 3, not including 3 itself. In interval notation, this can be written as (-∞, 3).

Hence, the inequality is x < 3.

To learn more about inequality equations, refer:

https://brainly.com/question/11897796

#SPJ3

1.5c - 6 = 7.5 i need help with this problem and i have no one to help me

Answers

Answer:

c=9

Step-by-step explanation:

Let's solve your equation step by step:

add 6 on both sidesdivide 1.5 from the product of 6 and 7.5 (aka 13.5)you will get Cput 9 in for C.[tex]1.5c-6=7.5[/tex]

1. [tex]1.5c-6+6=7.5+6[/tex]

1.5 [tex]1.5c=13.5[/tex]

2.[tex]\frac{1.5c}{1.5c}=\frac{13.5}{1.5c}[/tex]

3. C=9

1.5(9)-6=7.5

13.5-6=7.5

Please please help me with this question

Answers

Answer:

By definition definition a quadratic polynomial is a polynomial of degree two, the highest degree ter. is of second order.

A linear polynomial is defined by y=mx+c, the highest order is of degree 1.

If your x is raised to a fraction power then that is neither quadratic nor linear.

Write a two column proof of the statement. If 9k - 5 = 7k + 3, then k = 4.

Answers

Step-by-step explanation:

9k-5=7k+3

9k-7k=3+5

2k =8

k=4

associative property for 72.3+(-39.1)

Answers

The answer is I think now sure 33.2

A cashier has a total of 98 bills, made up of fives and fifties. The total value of the money is $2605. How much of each kind does he have?
He has ___ fives and ___ fifties.

Answers

Answer:

Step-by-step explanation:

x= number of fives

y= number of fifties

x+y=98

5x+50y=2605

Solving through elimination...

5x+5y=490

(5x+50y)-(5x+5y)=2605-490

45y=2115

y= 47

x+47=98

x=51

Which means he has 51 fives and 47 fifties

Hi I need help with this question please 20 points

Answers

Answer:

log 4

---------   that's the answer yw

   9

Step-by-step explanation:

-1.3 - 2 5/6 i need the answer plz help

Answers

-4 2/15 or -4.13, hopes that get you right

whats 186,282 rounded to the nearest thousand ?

Answers

Answer: 186,000

Step-by-step explanation:

You round the 282 down so that it is an even thousand. <3

186,282 rounded to the nearest thousand is 186,000.

To round 186,282 to the nearest thousand, we look at the digit in the thousands place, which is '6'.

If the digit in the hundreds place is 5 or greater, we round up. If it is less than 5, we round down.

In this case, since the digit in the hundreds place is '8', which is greater than 5, we round up.

To round up, we increase the digit in the thousands place by 1 and replace all the digits to the right with zeros.

Therefore, rounding 186,282 to the nearest thousand gives us 187,000.

To know more about nearest thousand, refer here:

https://brainly.com/question/29129004

#SPJ2

1 Which of these equations has infinite
many solutions
6(x+3)+x=7x+2+1
6(x+3)+x=7x+5
6(x+ 3) +x=6x+3+15
6(x+3)+x=7x+9+9

Answers

Answer:

Is, 6x+18+x=7x+2+1

7x+18=7x+2+1

what the power a when 5^a = 1/125

Answers

Answer:

a = -3

General Formulas and Concepts:

Algebra I

Solving Exponential EquationsExponential Property [Rewrite]: [tex]\displaystyle b^{-m} = \frac{1}{b^m}[/tex]

Step-by-step explanation:

Step 1: Define

Identify

[tex]\displaystyle 5^a = \frac{1}{125}[/tex]

Step 2: Solve for a

Rewrite:                                                                                                                [tex]\displaystyle 5^a = \frac{1}{5^3}[/tex]Rewrite [Exponential Property - Rewrite]:                                                       [tex]\displaystyle 5^a = 5^{-3}[/tex]Set:                                                                                                                    [tex]\displaystyle a = -3[/tex]

could someone please help answer this, please no spam, will give brainliest, thank you!
(homework help)

Answers

Answer:

[tex]x>-5[/tex]

Step-by-step explanation:

[tex]\frac{-1}{5} x+1<2[/tex]

Subtract 1 from both sides

[tex]\frac{-1}{5} x+1-1<2-1[/tex]

Simplify

[tex]\frac{-1}{5} x<1[/tex]

Multiply both sides by -1, which reverses the inequality also

[tex]\frac{-1}{5} x(-1)>1(-1)[/tex]

Simplify

[tex]\frac{1}{5} x>-1[/tex]

Multiply both sides by 5

[tex]5*\frac{1}{5} x>5(-1)[/tex]

Simplify

[tex]x>-5[/tex]

PLEASE HELP WITH PART B!!

can you also explain y and x, I dont understand part B >

Answers

A. I would suggest including all units in your answer. Both answers are correct but the unit is also important.

average rate of change:

($750 - $350) / (2014 - 2010) = $400 / (4 years) = $100 per year

initial value:

$350

B. Take 2010 to be year 0. Then for year x = 0, the fee is y = $350.

If the fee increases at a rate of $100 per year, this means that when x = 1, y = $350 + $100 = $450. So the graph of the function that models the dog-walking fee is a line that passes through the points (0, 350) and (1, 450), adn the slope of this line is

(450 - 350) / (1 - 0) = 100/1 = 100

which is the same as the average rate of change.

Using the point-slope formula for a line, the equation you want is then

y - 350 = 100 (x - 0)

y - 350 = 100x

y = 100x + 350

For brainily please help :/

Answers

Answer:

No, a triangle can not be constructed of 6, 2, and 3.

Step-by-step explanation:

For three line segments to be able to form any triangle you must be able to take any two sides, add their length and this sum be greater than the remaining side.

(g + 4) – 3g = 1 + g

Answers

Answer:

4-2g=1+g

4-1=g+2g

3=3g

G=1

Step-by-step explanation:

NO LINKS!!!!!!!!!!!!

Answers

Answer:

this is the answer thanks mark me as brainlist

Seven less than twice a
number is negative five.
What is the number?

Answers

Answer:

n = 1

Step-by-step explanation:

Represent the number by n.  Then "seven less than twice a number is -5 becomes 2n - 7 = -5

Next we must solve for n.  Adding 7 to both sides yields 2n = 2, so that

n = 1

what is the percent of games won if a team won 95 games and lost 65 games?

Answers

Answer:

The percent for games that were won is 59.375%.

Step-by-step explanation:

Other Questions
22/7 - 4/7 A)2 B)2 4/7 C)2 5/7 D)3 the movie theater offers a VIP member where you pay $75 for the VIP pass and then $4 per movie you see. People without the VIP pass that walk up to the movie theater pays $15 per movie they see A company ships bottles of olive oil to stores in boxes.The graph shows a proportional relationship betweenthe number of boxes shipped and the number ofbottles of olive oil shipped. About how many bottles areshipped in 8 boxes? With which statement would the with which statement would the author of the passage most likely agree?O Natural talent was the primary cause of Henry Ford's success.O Henry Ford shared his prosperity with others who were less fortunate.O Without an experience with failure, Henry Ford would have become arrogant.O Without Henry Ford's efforts, Americans would still be riding horses. 2 4^5 + 10 ( 5 2 ) - 10 2 = he conflict in this excerpt affects Jesse because he decides to avoid speaking publicly about the situation. confront the racist crowd about their remarks. only go to the meeting if Neto went with him. address the school during the morning announcements. What gas is dissolved into soft drinks such as soda which causes them to bubble when opened? A. Carbon dioxide B. Ethylene C. Hydrogen D. Argon the GCF of 30m^4 and 36m^6 is 6m^?the missing exponent is ____ How does Chekhov's use of the third-person limited point of view in "The Bet" affect the story?a- It allows him to reveal the lawyer's reasons for choosing certain subjects of study during his confinement.b- It allows him to spell out the terms of the bet.c- He is able to show the banker's worries about his possible defeat by the lawyer.d- He is able to include the essay in which the lawyer explains himself. NEED ANSWERS QUICK! WILL MARK BRAINLIEST!What is the m 12 in matrix M?M = 8 7 4 2 0 5a) 2b) 7c) 5d) 4 help!!! will give brainliest if correct !!!!! Joshua finds the perimeter of the following composite figure composed of a square and right triangle so that a braid may be cut around its outer edge. Determine what is wrong with his calculations and how to find the correct length. For my mathematics homework What single decimal multiplier would you use to decrease by 2% followed by a 7% decrease? Please can someone help me this is being timed name the macromolecules that are chosen as drug targets. HELPPPPPPPPPPPPPPPPP ASAPPPPP!!!! find the scale factors for the dilation of the small quadrilateral to the large a. 2b. 5c. 4d.3 What is the formula for Iron (3) Oxide? A projectile is launchedhorizontally from a clifftop at 16 m/s.Determine the valuesof the velocitycomponents at 1-second intervals oftime.